The OEIS mourns the passing of Jim Simons and is grateful to the Simons Foundation for its support of research in many branches of science, including the OEIS.
login
The OEIS is supported by the many generous donors to the OEIS Foundation.

 

Logo
Hints
(Greetings from The On-Line Encyclopedia of Integer Sequences!)
A091342 Given (1) f(h,j,a) = ( [ ((a/gcd(a,h)) / gcd(j+1,(a/gcd(a,h)))) * (h(j+1)) ] - [ ((a/gcd(a,h)) / gcd(j+1,(a/gcd(a,h)))) * (ja) ] ) / a then let (2) a(h) = d(h,j) = lcm( f(h,j,1) ... f(h,j,h) ). 3

%I #13 May 31 2022 06:50:25

%S 1,3,10,105,252,2310,25740,45045,680680,11639628,10581480,223092870,

%T 1029659400,2868336900,77636318760,4512611027925,4247163320400,

%U 4011209802600,140603459396400,133573286426580,5215718803323600

%N Given (1) f(h,j,a) = ( [ ((a/gcd(a,h)) / gcd(j+1,(a/gcd(a,h)))) * (h(j+1)) ] - [ ((a/gcd(a,h)) / gcd(j+1,(a/gcd(a,h)))) * (ja) ] ) / a then let (2) a(h) = d(h,j) = lcm( f(h,j,1) ... f(h,j,h) ).

%C Solves the following arithmetic problem. Let (3) q#(i/j)=(q+h) be defined thus: Given q, an unknown fraction, q=qd/d, of integer value (initially) and h equal to any desired integer value and j equal to any desired integer value and i equal to j(q+h)+h.

%C Let '#' denote the repeated addition of the denominator j to the denominator of q and the repeated addition of the numerator i to the numerator of q, each addition recursively replacing the prior q fractions numerator and denominator respectively. This results in a series of fractions of mostly non-integer values.

%C After the first three iterations for any case would result in the following fractions: 1) (qd+i) / (d+j) 2) ((qd+i)+i) / ((d+j)+j) 3) (((qd+i)+i)+i) / (((d+j)+j)+j)

%C The question is, what is the smallest initial denominator, d (of q=qd/d) that in the course of the repeated additions will result in fractions of integer value, where every integer, from the initial (q+1)...(q+h) will be formed?

%C For example, let q = 4, h = 5, so (q+h)=9 and j = 1, so that for i we have i = 14. So in terms of q#(i/j)=(q+h) we have 4#(14/1)=9.

%C In this sample, since h=5 and j=1 we get from (2) above that d(5,1) = 252 as the solution. Applying this solution, we see then that the initial numerator of q, or q*d, becomes 4*252 = 1008 and the initial denominator is 252. Alternatively, in terms of q#(i/j)=(q+h) we have (1008/252)#(14/1)=9. We see that the repeated additions yield:

%C 1) 1008 +14 and 252 +1 ~ 1022/253

%C 2) 1022 +14 and 253 +1 ~ 1036/254

%C ...

%C 27) 1372 +14 and 277 +1 ~ 1386/279

%C 28) 1386 +14 and 278 +1 ~ 1400/280 =5

%C ...

%C 63) 1876 +14 and 314 +1 ~ 1890/315 =6

%C ...

%C 108) 2506 +14 and 359 +1 ~ 2520/360 =7

%C ...

%C 168) 3346 +14 and 419 +1 ~ 3360/420 =8

%C ...

%C 252) 4522 +14 and 503 +1 ~ 4536/504 =9

%C Note that h=5 and j=1 were chosen so that d(5,1) = a(5) of the sequence. Also note that by the definition of q#(i/j)=(q+h) all answers are only a function of h and j.

%C Note also that q=qd/d can also be expressed as 0=0d/d and that any q#(i/j)=(q+h) can be expressed as 0#(i/j)=h [after adjustments are made to i]. For instance 4#(14/1)=9 is the equivalent of 0#(10/1)=5 in terms of d(h,j).

%C Interestingly: For any q#(i/j)=p and r#(s/j)=t then (q+r)#((i+s)/j)=(p+t). Also for any q#(i/j)=p then (qr)#((ir)/j)=(pr).

%C The sequence A025558 can be calculated from this formula when h = j, in otherwords using the sequence of d(1, 1)...d(n, n). i.e. a(7) = 735 = d(7,7) = lcm(49,21,35,7,21,7,1) a(8) = 2240 = d(8,8) = lcm(64,28,16,10,32,416,1)

%C Denominator of the sum of all elements of n X n Hilbert Matrix M[i,j] with alternate signs. M[i,j] = 1/(i+j-1)(i,j = 1..n). - _Alexander Adamchuk_, Apr 11 2006

%H Eric Weisstein's World of Mathematics, <a href="http://mathworld.wolfram.com/HilbertMatrix.html">Hilbert Matrix</a>.

%H Eric Weisstein's World of Mathematics, <a href="http://mathworld.wolfram.com/HarmonicNumber.html">Harmonic Number</a>.

%F a(n) = Denominator[Sum[Sum[(-1)^(i+j)*1/(i+j-1),{i,1,n}],{j,1,n}]]. - _Alexander Adamchuk_, Apr 11 2006

%e a(5) = lcm(9,4,7,3) = 252

%e a(7) = lcm(13,6,11,5,9,4,1) = 25740

%e a(10)= lcm(19,9,17,4,3,7,13,3,11,1) = 11639628

%e a(14)= lcm(27,13,25,6,23,11,3,5,19,9,17,4,15,1) = 2868336900

%e n=2: HilbertMatrix[n,n]

%e 1 1/2

%e 1/2 1/3

%e so a(2) = Denominator[(1 - 1/2 - 1/2 + 1/3)] = Denominator[1/3] = 3.

%e The n X n Hilbert matrix begins:

%e 1 1/2 1/3 1/4 1/5 1/6 1/7 1/8 ...

%e 1/2 1/3 1/4 1/5 1/6 1/7 1/8 1/9 ...

%e 1/3 1/4 1/5 1/6 1/7 1/8 1/9 1/10 ...

%e 1/4 1/5 1/6 1/7 1/8 1/9 1/10 1/11 ...

%e 1/5 1/6 1/7 1/8 1/9 1/10 1/11 1/12 ...

%e 1/6 1/7 1/8 1/9 1/10 1/11 1/12 1/13 ...

%t Denominator[Table[Sum[Sum[(-1)^(i+j)*1/(i+j-1),{i,1,n}],{j,1,n}],{n,1,40}]] (* _Alexander Adamchuk_, Apr 11 2006 *)

%Y Cf. A025558.

%Y Cf. A098118, A086881, A005249, A001008, A002805.

%K nonn,uned

%O 1,2

%A Scott C. Macfarlan (scottmacfarlan(AT)covance.com), Mar 01 2004

Lookup | Welcome | Wiki | Register | Music | Plot 2 | Demos | Index | Browse | More | WebCam
Contribute new seq. or comment | Format | Style Sheet | Transforms | Superseeker | Recents
The OEIS Community | Maintained by The OEIS Foundation Inc.

License Agreements, Terms of Use, Privacy Policy. .

Last modified May 21 13:13 EDT 2024. Contains 372736 sequences. (Running on oeis4.)